Đến nội dung

Hoang Tung 126 nội dung

Có 1000 mục bởi Hoang Tung 126 (Tìm giới hạn từ 05-06-2020)



Sắp theo                Sắp xếp  

#454216 $ \dfrac{xy}{ay+bx} = \dfrac{yz}...

Đã gửi bởi Hoang Tung 126 on 30-09-2013 - 13:50 trong Phương trình, hệ phương trình và bất phương trình

-Nếu x= thì y=0,z=0$= >$ Biểu thức vô nghĩa

$= > x,y,z$ đều khác 0.

Từ hệ phương trình ta có :$\frac{a}{x}+\frac{b}{y}=\frac{b}{y}+\frac{c}{z}=\frac{c}{z}+\frac{a}{x}=\frac{a^2+b^2+c^2}{x^2+y^2+z^2}= > 2(\frac{a}{x}+\frac{b}{y}+\frac{c}{z})=3(\frac{a^2+b^2+c^2}{x^2+y^2+z^2})< = > \frac{a}{x}+\frac{b}{y}+\frac{c}{z}=\frac{3}{2}.\frac{a^2+b^2+c^2}{x^2+y^2+z^2}= > \frac{a}{x}=\frac{b}{y}=\frac{c}{z}=\frac{3}{2}.\frac{a^2+b^2+c^2}{x^2+y^2+z^2}-\frac{a^2+b^2+c^2}{x^2+y^2+z^2}=\frac{a^2+b^2+c^2}{2(x^2+y^2+z^2)}$

Đặt $\frac{x}{a}=\frac{y}{b}=\frac{z}{c}=t= > x=at,y=bt,z=ct= > \frac{a}{x}=\frac{a}{at}=\frac{a^2+b^2+c^2}{2t^2(a^2+b^2+c^2)}=\frac{1}{2t^2}= > \frac{1}{t}=\frac{1}{2t^2}= > t=\frac{1}{2}= > x=\frac{a}{2},y=\frac{b}{2},z=\frac{c}{2}$




#474199 (1+sinx)(2sin2x+1-2sinx-2cosx) + $(sinx+cosx)^{4}-2(sinx+cosx)...

Đã gửi bởi Hoang Tung 126 on 31-12-2013 - 18:43 trong Phương trình, Hệ phương trình Lượng giác

Bạn giải giúp mình đc k??

Để mình xem đã




#474190 (1+sinx)(2sin2x+1-2sinx-2cosx) + $(sinx+cosx)^{4}-2(sinx+cosx)...

Đã gửi bởi Hoang Tung 126 on 31-12-2013 - 18:27 trong Phương trình, Hệ phương trình Lượng giác

GIẢI PT

(1+sinx)(2sin2x+1-2sinx-2cosx) + $(sinx+cosx)^{4}-2(sinx+cosx)^{2}$ = 1

Mình sửa đề cho bạn lại như sau : $(1+sinx)(2sin2x+1-2sinx-2cosx)+(sinx+cosx)^4-2(sinx+coxx)^2=1$




#474195 (1+sinx)(2sin2x+1-2sinx-2cosx) + $(sinx+cosx)^{4}-2(sinx+cosx)...

Đã gửi bởi Hoang Tung 126 on 31-12-2013 - 18:39 trong Phương trình, Hệ phương trình Lượng giác

bạn sửa lại giùm mình cái cuối k có mũ 2 cho mình với!!!!

$(1+sinx)(2sin2x+1-2sinx-2cosx)+(sinx+cosx)^4-2(sinx+cox)=1$




#460668 (a+b)(c+d) < ab+ cd

Đã gửi bởi Hoang Tung 126 on 29-10-2013 - 16:50 trong Bất đẳng thức và cực trị

Bài 1: Giả sử $a< 0$.Do $a+b+c> 0= > b+c> 0= > a(b+c)< 0$.Do $ab+bc+ac> 0= > bc> 0= > abc< 0$(vô lý do $abc> 0$)

$= > a> 0$.Tương tự $b> 0,c> 0$




#474425 (a-3)(b-4)=12;tìm cặp a,b sao cho ${\frac{1}{a^...

Đã gửi bởi Hoang Tung 126 on 01-01-2014 - 12:39 trong Bất đẳng thức và cực trị

trong tất cả các cặp số nguyên dương a,b thoả mãn (a-3)(b-4)=12

hãy tìm cặp a,b sao cho ${\frac{1}{a^2}+\frac{1}{b^2}}_{max}$ 

thank nhé :icon6:  :icon6:  :icon6:  :icon6:

Để $\frac{1}{a^2}+\frac{1}{b^2}$ Max $< = > a,b$ Min .

Ta có :$(a-3)(b-4)=12=1.12=2.6=3.4$

Đến đây thử các TH là ra




#454241 )$sin^{6}\alpha +cos^{6}\alpha +3(sin^{2}\alpha )\ti...

Đã gửi bởi Hoang Tung 126 on 30-09-2013 - 15:40 trong Hình học

Bài 2: Ta có :$cotB+cotC=\frac{BH}{AH}+\frac{CH}{AH}=\frac{BH+CH}{AH}=\frac{BC}{AH}= > \frac{BC}{cotB+cotC}=\frac{BC}{\frac{BC}{AH}}=AH$(đpcm)




#477555 . Chứng minh bất đẳng thức sau: $a+b+c-abc\leq 4$

Đã gửi bởi Hoang Tung 126 on 16-01-2014 - 16:56 trong Bất đẳng thức và cực trị

Theo bdt Bunhiacopxki có :$(a+b+c-abc)^2=\left [ a(1-bc)+(b+c) \right ]^2\leq (a^2+(b+c)^2)((1-bc)^2+1^2)=(a^2+b^2+c^2+2bc)(b^2c^2-2bc+2)=(1+2bc)(b^2c^2-2bc+2)$

Do đó ta cần CM :$(3+2bc)(b^2c^2-2bc+2)\leq 16< = > 2b^3c^3-4b^2c^2+4bc+3b^2c^2-6bc+10\leq 0< = > 2b^3c^3-b^2c^2-2bc+10\leq 0$




#477736 . Chứng minh bất đẳng thức sau: $a+b+c-abc\leq 4$

Đã gửi bởi Hoang Tung 126 on 17-01-2014 - 20:41 trong Bất đẳng thức và cực trị

Bạn ơi, thế cứ biết nhiều bất đẳng thức hay là làm được nhiều bài tập à!

ừ tùy vào từng bài thôi không phải bài này cũng dùng được đâu




#477565 . Chứng minh bất đẳng thức sau: $a+b+c-abc\leq 4$

Đã gửi bởi Hoang Tung 126 on 16-01-2014 - 17:46 trong Bất đẳng thức và cực trị

Dựa vào đâu mà bạn biết cách chứng minh bài này thế!

Chỉ dựa vào đk và bdt Bunhiacopxki thôi




#477812 .CMR: $\frac{AB+BC+CA}{4}<R+r$

Đã gửi bởi Hoang Tung 126 on 18-01-2014 - 07:46 trong Bất đẳng thức và cực trị

Cho $\Delta ABC$ có R là bán kính đường tròn ngoại tiếp, r là bán kính đường tròn nội tiếp .CMR: $\frac{AB+BC+CA}{4}<R+r$

Đặt $AB=c,BC=a,CA=b$

Ta có :$R=\frac{abc}{4S}=\frac{abc}{4\sqrt{p(p-a)(p-b)(p-c)}},r=\frac{2S}{a+b+c}=\frac{2\sqrt{p(p-a)(p-b)(p-c)}}{a+b+c}$

Đặt $p-a=x,p-b=y,p-c=z= > a=y+z,b=x+z,c=x+y,a+b+c=2(x+y+z)$

BĐT $< = > \frac{\sum x}{2}< \frac{(x+y)(y+z)(x+z)}{4\sqrt{xyz(\sum x)}}+\frac{\sqrt{xyz(\sum x)}}{\sum x}< = > 2(\sum x)^2\sqrt{xyz(\sum x)}\leq (x+y)(y+z)(x+z)+4xyz(\sum x)$

Nhưng bđt này luôn đúng do theo AM-GM có :

  $(x+y)(y+z)(x+z)(\sum x)+4xyz(\sum x)=(\sum x)((x+y)(y+z)(x+z)+4xyz)> (\sum x)((x+y)(y+z)(x+z)+xyz)=(\sum x)(\sum x)(\sum xy)=(\sum x)^2(\sum xy)\geq (\sum x)^2\sqrt{3xyz(\sum x)}=(\sum x)^2.\frac{3}{\sqrt{3}}\sqrt{xyz(\sum x)}> (\sum x)^2.\frac{2}{\sqrt{3}}\sqrt{xyz(\sum x)}$




#486432 :$\frac{MA}{BC}+\frac{MB}{A...

Đã gửi bởi Hoang Tung 126 on 12-03-2014 - 15:30 trong Hình học

Cho tam giác ABC có độ dài 3 cạnh là a,b,c và M là điểm bất kì .

CMR :$\frac{MA}{BC}+\frac{MB}{AC}+\frac{MC}{AB}\geq \frac{BC+CA+AB}{MA+MB+MC}$




#484610 :$\left\{\begin{matrix} x^2+y^2=1\...

Đã gửi bởi Hoang Tung 126 on 24-02-2014 - 18:43 trong Phương trình - Hệ phương trình - Bất phương trình

Mình vừa mới tìm được một bài khá hay .

  Giải hệ phương trình :$\left\{\begin{matrix} x^2+y^2=1\\x^5+y^5+x^3y^5+y^3x^5=1+x^2y^2 \end{matrix}\right.$




#485117 :$(LST)$ chia đôi đoạn EF.

Đã gửi bởi Hoang Tung 126 on 28-02-2014 - 14:19 trong Hình học

Bài toán :Cho tam giác ABC .Đường tròn $(O)$ bất kì đi qua B,C.Đường tròn ngoại tiếp tam giác ABC cất BC tại E,F và cắt giao điểm đường trung trực AB,AC,BC tại L,S,T. CMR :$(LST)$ chia đôi EF




#482509 :$4a^3+4a^2+4a+1$ là số chính phương.

Đã gửi bởi Hoang Tung 126 on 11-02-2014 - 10:59 trong Số học

Bài toán:Tìm các số tự nhiên a sao cho số :$4a^3+4a^2+4a+1$ là số chính phương.




#472458 :$A_{1}A_{2},B_{1}B_{2},C_{...

Đã gửi bởi Hoang Tung 126 on 23-12-2013 - 15:40 trong Hình học

Bài toán :

   Cho $\Delta ABC$ nội tiếp $(O)$ và $M$ là điểm bất kì nằm trong tam giác .Nối $AM,BM,CM$ cắt đường tròn $(O)$ tại $A_{1},B_{1},C_{1}$. Gọi $A_{2},B_{2},C_{2}$ theo thứ tự là tâm đường tròn ngoại tiếp $\Delta BMC,\Delta AMC,\Delta AMB$.

   CMR :$A_{1}A_{2},B_{1}B_{2},C_{1}C_{2}$ đồng quy tại một điểm nằm trên $(O)$




#472648 :$A_{1}A_{2},B_{1}B_{2},C_{...

Đã gửi bởi Hoang Tung 126 on 24-12-2013 - 14:11 trong Hình học

 Các bạn chịu khó vẽ hình :

Ta có :$(A_{1}C,A_{1}B_{1})\equiv (BC,BB_{1})(mod \pi )\equiv (BC,BM)(mod \pi )\equiv \frac{1}{2}(\overrightarrow{A_{2}C},\overrightarrow{A_{2}M})(mod \pi )\equiv (A_{2}C,A_{2}B_{2})(mod \pi )$

Tương tự :$(B_{1}C,B_{1}A_{1})\equiv (B_{2}C,B_{2}A_{2})(mod \pi )$

$= > \Delta A_{1}B_{1}C$ và $\Delta A_{2}B_{2}C$ đồng dạng cùng hướng .

$= > \Delta A_{1}A_{2}C$ và $\Delta B_{1}B_{2}C$ đồng dạng cùng hướng .

 Giả sử $A_{1}A_{2}$ cắt $(O)$ tại $P$

Ta có :$(PA_{1},B_{2}B_{1})\equiv (A_{2}A_{1},B_{2}B_{1})\equiv (CA_{1},CB_{1})\equiv (PA_{1},PB_{1})(mod \pi )= > B_{1}B_{2}$ đi qua điểm P 

Tương tự $C_{1}C_{2}$ đi qua P 

   Vậy $A_{1}A_{2},B_{1}B_{2},C_{1}C_{2}$ đồng quy tại điểm P




#511833 [TSĐH 2014] Đề thi khối B

Đã gửi bởi Hoang Tung 126 on 09-07-2014 - 11:36 trong Thi TS ĐH


ĐỀ THI TUYỂN SINH ĐẠI HỌC MÔN TOÁN KHỐI B NĂM 2014

---------------------

Câu 1: (2,0 điểm) Cho hàm số $y = {x^3} - 3mx + 1$ (1), với $m$ là tham số thực.
a. Khảo sát sự biến thiên và vẽ đồ thị của hàm số $(1)$ khi $m=1$.
b. Cho điểm $A(2;3)$. Tìm $m$ để đồ thị hàm số $(1)$ có hai điẻm cực trị $B$ và $C$ sao cho tam giác $ABC$ cân tại $A$.

Câu 2: (1,0 điểm) Giải phương trình $\sqrt 2 \left( {\sin x - 2\cos x} \right) = 2 - \sin 2x$.

Câu 3: (1,0 điểm) Tính tích phân $\int\limits_1^2 {\frac{{{x^2} + 3x + 1}}{{{x^2} + x}}dx} $

Câu 4: (1,0 điểm)
a. Cho số phức $z$ thỏa mã điều kiện $2z + 3\left( {1 - i} \right)\overline z = 1 - 9i$. Tính môđun của $z$.

b. Để kiểm tra chất lượng sản phân từ một công ty sữa, người ta phải gửi đến bộ phận kiểm j nghiệm 5 hộp sữa cam, 4 hộp sữa dâu và 3 hộp sữa nho. Bộ phận kiểm nghiệm chọn ngẫu nhiên 3 hộp sữa để phân tích nẫu. Tính xác suất để 3 hộp sữa được chọn có cả 3 loại.

Câu 5: (1,0 điểm) Trong không gian với hệ tọa độ $Oxyz$, cho điểm $A(1;0;-1)$ và đường thẳng $d:\frac{{x - 1}}{2} = \frac{{y + 1}}{2} = \frac{z}{{ - 1}}$. Viết phương trình mặt phẳng qua $A$ và vuông góc với $d$. Tìm tọa độ hình chiếu vuông góc của $A$ trên $d$.

Câu 6: (1,0 điểm) Cho lăng trụ $ABC.A'B'C'$ có đấy là tam giác đều cạnh $a$. Hình chiếu vuông góc của $A'$ trên mặt phẳng $(ABC)$ là trung điểm của cạnh $AB$, góc giữa đường thẳng $A'C$ và mặt đáy bằng $60^\circ $. Tính theo $a$ thể tích của khối lăng trụ $ABC.A'B'C'$ và khoảng cách từ điểm $B$ đến mặt phẳng $(ACC'A')$.

Câu 7: (1,0 điểm) Trong mặt phẳng với hệ tọa độ $Oxy$, cho hình bình hành $ABCD$. ĐIểm $M(-3;0)$ là trung điểm của cạnh $AB$, điểm $H(0;-1)$ là hình chiếu vuông góc của $B$ trên $AD$ và điểm $G\left( {\frac{4}{3};3} \right)$ là trọng tâm tam gáic $BCD$. Tìm tọa độ các điểm $B$ và $D$

Câu 8: (1,0 điểm) Giải hệ phương trình $\left\{\begin{matrix} \left( {1 - y} \right)\sqrt {x - y} + x = 2 + \left( {x - y - 1} \right)\sqrt y \\ 2{y^2} -3x+6y + 1=2\sqrt {x - 2y} - \sqrt {4x - 5y - 3} \end{matrix}\right.\left( {x,y \in R} \right)$

Câu 9: (1,0 điểm) Cho các số thực $a,b,c$ không âm thỏa mã điều kiện $(a+b)c>0$. Tìm giá trị nhỏ nhất của biểu thức

$P = \sqrt {\frac{a}{{b + c}}} + \sqrt {\frac{b}{{a + c}}} + \frac{c}{{2\left( {a + b} \right)}}$


_HẾT_

Câu 9:(Câu này không khó hơn đề A)
Theo AM-GM có :$P=\sqrt{\frac{a}{b+c}}+\sqrt{\frac{b}{a+c}}+\frac{c}{2(a+b)}=\frac{a}{\sqrt{a(b+c)}}+\frac{b}{\sqrt{b(a+c)}}+\frac{c}{2(a+b)}\geq \frac{a}{\frac{a+b+c}{2}}+\frac{b}{\frac{a+b+c}{2}}+\frac{c}{2(a+b)}=\frac{2(a+b)}{a+b+c}+\frac{c}{2(a+b)}=\frac{2(a+b)}{a+b+c}+(\frac{c}{2(a+b)}+\frac{1}{2})-\frac{1}{2}=\frac{2(a+b)}{a+b+c}+\frac{a+b+c}{2(a+b)}-\frac{1}{2}\geq 2\sqrt{\frac{2(a+b)(a+b+c)}{(a+b+c).2(a+b)}}-\frac{1}{2}=2-\frac{1}{2}=\frac{3}{2}= > P\geq \frac{3}{2}$
Dấu =xảy ra khi $a=0,b=a+c,4(a+b)^2=(a+b+c)^2< = > a=0,b=c$
Mod : làm câu nào thì trích dẫn câu đó thôi



#511838 [TSĐH 2014] Đề thi khối B

Đã gửi bởi Hoang Tung 126 on 09-07-2014 - 11:58 trong Thi TS ĐH

Câu 9:(Câu này không khó hơn đề A)
Theo AM-GM có :$P=\sqrt{\frac{a}{b+c}}+\sqrt{\frac{b}{a+c}}+\frac{c}{2(a+b)}=\frac{a}{\sqrt{a(b+c)}}+\frac{b}{\sqrt{b(a+c)}}+\frac{c}{2(a+b)}\geq \frac{a}{\frac{a+b+c}{2}}+\frac{b}{\frac{a+b+c}{2}}+\frac{c}{2(a+b)}=\frac{2(a+b)}{a+b+c}+\frac{c}{2(a+b)}=\frac{2(a+b)}{a+b+c}+(\frac{c}{2(a+b)}+\frac{1}{2})-\frac{1}{2}=\frac{2(a+b)}{a+b+c}+\frac{a+b+c}{2(a+b)}-\frac{1}{2}\geq 2\sqrt{\frac{2(a+b)(a+b+c)}{(a+b+c).2(a+b)}}-\frac{1}{2}=2-\frac{1}{2}=\frac{3}{2}= > P\geq \frac{3}{2}$
Dấu =xảy ra khi $a=0,b=a+c,4(a+b)^2=(a+b+c)^2< = > a=0,b=c$
Mod : làm câu nào thì trích dẫn câu đó thôi

Nhưng trích dẫn từng câu kiểu gì vậy anh




#511901 [TSĐH 2014] Đề thi khối D

Đã gửi bởi Hoang Tung 126 on 09-07-2014 - 17:29 trong Thi TS ĐH

Câu 8: ĐK:$x\geq -2$

BPT $< = > x^2+2x-8+(x+1)(2-\sqrt{x+2})+(x+6)(3-\sqrt{x+7})\leq 0< = > (x-2)(x+4-\frac{x+1}{2+\sqrt{x+2}}-\frac{x+6}{3+\sqrt{x+7}})\leq 0$

Mặt khác $x+4-\frac{x+1}{2+\sqrt{x+2}}-\frac{x+6}{3+\sqrt{x+7}}\geq \frac{x+6}{3}-\frac{x+6}{3}+(\frac{2x+3}{3}-\frac{x+1}{2+\sqrt{x+2}})+1> 0= > x-2\leq 0= > -2\leq x\leq 2$




#475171 [VMO 2014] Ngày 1 - BÀI 4 - HÌNH HỌC PHẲNG

Đã gửi bởi Hoang Tung 126 on 04-01-2014 - 09:46 trong Hình học

Mình làm theo cách của mình .Hình vẽ của bạn http://diendantoanho...77-blackselena/ và Nối PK cắt đường tròn (O) tại G ,cắt AD tại Q

Câu a: Ta có :$IK=IC=IB$$= > I$ là tâm đường tròn $(BKC)= > \widehat{KIC}=2\widehat{KBC}=2\widehat{DBC}=2\widehat{FIC}= > \widehat{KIF}=\widehat{FIC},IK=IC= > KF=FC$

Tương tự $KE=BE$

Do đó EF là đường trung bình tam giác $BKC= > EF=\frac{BC}{2}$

Câu b: Do M thuộc trung trực KC $= > \widehat{MKC}=\widehat{MCK}=\widehat{MCA}=\widehat{CAD}=\widehat{KBC}$(Do $CM$ song song AD )(1)

 Mà theo tính chất góc nội tiếp có :$\widehat{MKP}=\widehat{NKP}=\widehat{PBC}$(2)

Từ (1),(2) $= > \widehat{AKG}=\widehat{PKC}=\widehat{KBP}=\widehat{DBP}=\widehat{PGD}= > AK$ song song $DG$(3)

$= > \widehat{GDK}=\widehat{AKD}=\widehat{ADK}$

Mà $ABDG$ là tứ giác nội tiếp nên ABDG là hình thang cân hay  DG=AB=AK(4)

Từ (3) và (4) dẫn tới AKDG là hình bình hành hay Q là trung điểm của AD

 Suy ra PK đi qua trung điểm Q của AD

  (Chắc các men KHTN là ngon bài này rùi......)




#474947 [VMO 2014] Ngày 1 - Bài 1 - DÃY SỐ

Đã gửi bởi Hoang Tung 126 on 03-01-2014 - 12:06 trong Dãy số - Giới hạn

Sao đề thi năm nay lại không có HPT nhỉ




#475736 [VMO 2014] Ngày 2 - Bài 5 - Hình học phẳng

Đã gửi bởi Hoang Tung 126 on 06-01-2014 - 17:31 trong Hình học

 Mình xin giới thiệu cách làm của THCS ,Hình vẽ như của bạn http://diendantoanho...77-blackselena/

Câu b:Vẽ 2 tiếp tuyến tại B và C của đường tròn $(O)$ cắt nhau tại $I$ .Do B,C cố định nên $I$ cố định .Ta sẽ CM: AF đi qua điểm $I$ cố định.

Gọi giao điểm của $AI$ với đường tròn $(O)$ là $F'$. Ta sẽ CM :$F\equiv F'$

-Do $AO=OB,AN=NB= > ON$ là trung trực đoạn $AM$ $= > ON$ vuông góc AB, Tương tự OM vuông góc AN 

Do đó O là trực tâm tam giác AMN nên AO vuông góc MN. Do 2 đường tròn tâm M và tâm N cắt nhau tại A và K nên MN là trung trực AK hay MN vuông góc AK :

 Từ đó suy ra A,O,K thẳng hàng .

Gọi D là giao điểm của OI và BC .Ta có :$IB^2=ID.IO=IF'.IA$ nên tứ giác $AODF'$ nội tiếp hay $\widehat{KOD}=\widehat{DF'A}$(1)

Do $\widehat{ODE}+\widehat{EAO}=90+90=180$ nên tứ giác AODE nội tiếp hay $\widehat{ODK}=\widehat{AED}$(2)

- Từ (1) ,(2) $= > \widehat{DF'A}=\widehat{AED}$ hay tứ giác $AEF'D$ nội tiếp .Từ đó $F'$ là giao điểm của 2 đường tròn $(AED),(ABC)$

$= > F\equiv F'$ $= > AF$ đi qua điểm I cố định 




#475521 [VMO 2014] Ngày 2 - Bài 6 - Đại số

Đã gửi bởi Hoang Tung 126 on 05-01-2014 - 18:07 trong Bất đẳng thức - Cực trị

Mình giới thiệu cách của mình :

 Ta có :$\sum \frac{x^3y^4z^3}{(x^4+y^4)(xy+z^2)^3}\leq \sum \frac{x^3y^4z^3}{\frac{(x+y)(x^3+y^3)}{2}.8xy.z^3.\sqrt{xy}}=\sum \frac{x^2y^3}{4(x+y)(x^3+y^3)\sqrt{xy}}\leq \sum \frac{x^2y^3}{8xy(x^3+y^3)}=\sum \frac{xy^2}{8(x^3+y^3)}\leq \sum \frac{xy^2}{4(x+y)(x^2+y^2)}\leq \sum \frac{xy^2}{8\sqrt{xy}.(x^2+y^2)}\leq \sum \frac{xy^2}{8\sqrt{xy}.\sqrt{2xy(x^2+y^2)}}= \frac{1}{8\sqrt{2}}.\sum \sqrt{\frac{y^2}{x^2+y^2}}\leq \frac{1}{8\sqrt{2}}.\frac{3}{\sqrt{2}}=\frac{3}{16}$(ĐPCM)

 Dấu = xảy ra tại x=y=z




#475231 [VMO 2014] Ngày 2 - Bài 6 - Đại số

Đã gửi bởi Hoang Tung 126 on 04-01-2014 - 14:10 trong Bất đẳng thức - Cực trị

Bài 6 (7 điểm). Tìm giá trị lớn nhất của biểu thức
$$T=\frac{x^3y^4z^3}{(x^4+y^4)(xy+z^2)^3}+\frac{y^3z^4x^3}{(y^4+z^4)(yz+x^2)^3}+\frac{z^3x^4y^3}{(z^4+x^4)(zx+y^2)^3}$$
với $x,y,z$ là các số thực dương.

Ta có : $\sum \frac{x^3y^4z^3}{(x^4+y^4)(xy+z^2)^3}\leq \sum \frac{x^3y^4z^3}{2x^2y^2.(xy+z^2)^3}=\sum \frac{xy^2z^3}{2(xy+z^2)^3}=\sum \frac{xy.z^2.yz}{2(xy+z^2)^3}\leq \frac{\frac{(xy+z^2)^2.yz}{4}}{2(xy+z^2)^3}=\frac{yz}{8(xy+z^2)}$

Do đó ta sẽ CM :$\sum \frac{yz}{xy+z^2}\leq \frac{3}{2}< = > \sum \frac{1}{\frac{x}{z}+\frac{z}{y}}\leq \frac{3}{2}$

Đặt $\frac{y}{x}=c,\frac{z}{y}=a,\frac{x}{z}=b$

BĐT $< = > \sum \frac{1}{a+b}\leq \frac{3}{2}$ với $abc=1$

Thế nhưng đây là bđt quen thuộc .

Do đó A Max= $\frac{3}{16}< = > x=y=z$